2
$\begingroup$

Given a bijective function $F: \mathbb{F}_2^n \rightarrow \mathbb{F}_2^n$.

The entry of the Difference Distribution Table (DDT) at row $\alpha$ and column $\beta$ is defined as

$$DDT_{F}(\alpha,\beta) = \delta_F(\alpha, \beta) = |\{ x \in \mathbb{F}_2^n | F(x) + F(x+\alpha) = \beta \}|$$

I need to show that

$$DDT_{F}(\alpha,\beta) = DDT_{F^{-1}}(\beta, \alpha)$$

with

$$DDT_{F^{-1}}(\beta, \alpha) = \delta_{F^{-1}}(\beta, \alpha) = |\{ x \in \mathbb{F}_2^n | F^{-1}(x) + F^{-1}(x+\beta) = \alpha \}|$$

Unfortunately, I do not know how to start to show the equality, but I assume substitution is the way to go. Can you please give me a hint?


I tried the same approach I used to show that this symmetric property does also hold for the Linear Approximation Table (LAT), i.e.

$$LAT_{F}(\alpha,\beta) = LAT_{F^{-1}}(\beta, \alpha)$$

with

$$LAT_F(\alpha,\beta) = \widehat{F}(\alpha,\beta) = \sum_{x \in \mathbb{F}_2^n} (-1)^{<\alpha,x> + <\beta,F(x)>}$$

$$LAT_{F^{-1}}(\beta,\alpha) = \widehat{F^{-1}}(\beta,\alpha) = \sum_{x \in \mathbb{F}_2^n} (-1)^{<\beta,x> + <\alpha,F^{-1}(x)>}$$

For the LAT, the trick was to substitute. Since $F$ is a permutation and the sum goes over all elements in $\mathbb{F}_2^n$, substitution only changes the order in which the elements in $\mathbb{F}_2^n$ are processed.

$$x = F(y)$$

This leads to:

$$\widehat{F^{-1}}(\beta,\alpha) = \sum_{x \in \mathbb{F}_2^n} (-1)^{<\beta,x> + <\alpha,F^{-1}(x)>} = \sum_{y \in \mathbb{F}_2^n} (-1)^{<\beta,F(y)> + <\alpha,y>} = \widehat{F}(\alpha,\beta)$$

Unfortunately, substituting $x = F(y)$ did not solve my issue with the DDT.

$\endgroup$
0

1 Answer 1

2
$\begingroup$

The proof for DDT is shown as following:

given: $$y=F(x) ;\\ x=F^{-1}(y) \> \> (1) $$ The forward DDT is $$F(x) + F(x+\alpha) = \beta \>\> (2)$$ substituting (1) into (2) $$F(x) + F(F^{-1}(y)+\alpha) = \beta $$

$$y+ \beta +F(F^{-1}(y)+\alpha) =0$$

re-arranging and applying inverse function $$F^{-1}(y+ \beta) +F^{-1}((F(F^{-1}(y)+\alpha))=0 $$

$$F^{-1}(y+ \beta) +F^{-1}(y) = \alpha$$

Therefore ,

$$DDT_{F^{-1}}(\beta, \alpha) = \delta_{F^{-1}}(\beta, \alpha) = |\{ x \in \mathbb{F}_2^n | F^{-1}(x) + F^{-1}(x+\beta) = \alpha \}|$$

The same concept applied to LAT proof.

$\endgroup$
1
  • $\begingroup$ Applying $F^{-1}()$ after the substitution was the part I didn't get. Now the solution seems to be pretty obvious. Thank you for helping me out. $\endgroup$
    – Florian
    Commented May 29, 2019 at 19:41

Your Answer

By clicking “Post Your Answer”, you agree to our terms of service and acknowledge you have read our privacy policy.

Start asking to get answers

Find the answer to your question by asking.

Ask question

Explore related questions

See similar questions with these tags.